Difference between revisions of "1993 AHSME Problems/Problem 21"

 
(9 intermediate revisions by 2 users not shown)
Line 1: Line 1:
Let a_1,a_2,...,a_k be a finite arithmetic sequence with a_4 +a_7+a_10 = 17 and a_4+a_5+...+a_13 +a_14 = 77. if a_k = 13, then k =
+
== Problem ==
  
(A) 16     (B) 18     (C) 20     (D) 22     (E) 24
+
Let <math>a_1,a_2,\cdots,a_k</math> be a finite arithmetic sequence with <math>a_4 +a_7+a_{10} = 17</math> and <math>a_4+a_5+\cdots+a_{13} +a_{14} = 77</math>.
 +
 
 +
If <math>a_k = 13</math>, then <math>k =</math>
 +
 
 +
<math>\text{(A) } 16\quad
 +
\text{(B) } 18\quad
 +
\text{(C) } 20\quad
 +
\text{(D) } 22\quad
 +
\text{(E) } 24</math>
 +
 
 +
== Solution ==
 +
Note that <math>a_7-3d=a_4</math> and <math>a_7+3d=a_{10}</math> where <math>d</math> is the common difference, so <math>a_4+a_7+a_{10}=3a_7=17</math>, or <math>a_7=\frac{17}{3}</math>.
 +
 
 +
Likewise, we can write every term in the second equation in terms of <math>a_9</math>, giving us <math>11a_9=77\implies a_9=7</math>.
 +
 
 +
Then the common difference is <math>\frac{2}{3}</math>. Then <math>a_k-a_9=13-7=6=9\cdot\frac{2}{3}</math>.
 +
 
 +
This means <math>a_k</math> is <math>9</math> terms after <math>a_9</math>, so <math>k=18\implies\boxed{B}</math>
 +
== See also ==
 +
{{AHSME box|year=1993|num-b=20|num-a=22}} 
 +
 
 +
[[Category: Intermediate Algebra Problems]]
 
{{MAA Notice}}
 
{{MAA Notice}}

Latest revision as of 17:49, 26 August 2017

Problem

Let $a_1,a_2,\cdots,a_k$ be a finite arithmetic sequence with $a_4 +a_7+a_{10} = 17$ and $a_4+a_5+\cdots+a_{13} +a_{14} = 77$.

If $a_k = 13$, then $k =$

$\text{(A) } 16\quad \text{(B) } 18\quad \text{(C) } 20\quad \text{(D) } 22\quad \text{(E) } 24$

Solution

Note that $a_7-3d=a_4$ and $a_7+3d=a_{10}$ where $d$ is the common difference, so $a_4+a_7+a_{10}=3a_7=17$, or $a_7=\frac{17}{3}$.

Likewise, we can write every term in the second equation in terms of $a_9$, giving us $11a_9=77\implies a_9=7$.

Then the common difference is $\frac{2}{3}$. Then $a_k-a_9=13-7=6=9\cdot\frac{2}{3}$.

This means $a_k$ is $9$ terms after $a_9$, so $k=18\implies\boxed{B}$

See also

1993 AHSME (ProblemsAnswer KeyResources)
Preceded by
Problem 20
Followed by
Problem 22
1 2 3 4 5 6 7 8 9 10 11 12 13 14 15 16 17 18 19 20 21 22 23 24 25 26 27 28 29 30
All AHSME Problems and Solutions

The problems on this page are copyrighted by the Mathematical Association of America's American Mathematics Competitions. AMC logo.png